LSAT and Law School Admissions Forum

Get expert LSAT preparation and law school admissions advice from PowerScore Test Preparation.

User avatar
 Dave Killoran
PowerScore Staff
  • PowerScore Staff
  • Posts: 5853
  • Joined: Mar 25, 2011
|
#85195
Complete Question Explanation
(The complete setup for this game can be found here: lsat/viewtopic.php?f=332&t=3599)

The correct answer choice is (A)

If J evaluates H during week 3, and R evaluates G during week 6, the initial setup appears as follows:

G3-Q12-d1.png

In order to conform to the first rule, R must then evaluate H during week 4, forcing F to be the textbook R evaluates during week 2. J must then evaluate F during week 1:

G3-Q12-d2.png

Consequently, answer choice (A) is correct.
You do not have the required permissions to view the files attached to this post.

Get the most out of your LSAT Prep Plus subscription.

Analyze and track your performance with our Testing and Analytics Package.